What is the limit of Gamma function as n approaches infinity?

That is pretty much her solution. However, I can actually follow yours better. I made an approximation that was not justified.Thanks,Chris
  • #1
kq6up
368
13

Homework Statement



Find the limit of: ##\frac { \Gamma (n+\frac { 3 }{ 2 } ) }{ \sqrt { n } \Gamma (n+1) } ## as ##n\rightarrow \infty ##.

Homework Equations



##\Gamma (p+1)=p^{ p }e^{ -p }\sqrt { 2\pi p } ##

The Attempt at a Solution



Mathematica and wolfram Alpha gave the limit as 1. My solution was ##\frac{1}{\sqrt{e}}##. My work is here https://plus.google.com/u/0/1096789...6080238892798007794&oid=109678926107781868876 Sorry about the photo, but my school is about to set the alarm, and I need to get out of here. Hopefully it is visible.

Thanks,
Chris
 
Physics news on Phys.org
  • #2
kq6up said:

Homework Statement



Find the limit of: ##\frac { \Gamma (n+\frac { 3 }{ 2 } ) }{ \sqrt { n } \Gamma (n+1) } ## as ##n\rightarrow \infty ##.

Homework Equations



##\Gamma (p+1)=p^{ p }e^{ -p }\sqrt { 2\pi p } ##

The Attempt at a Solution



Mathematica and wolfram Alpha gave the limit as 1. My solution was ##\frac{1}{\sqrt{e}}##. My work is here https://plus.google.com/u/0/1096789...6080238892798007794&oid=109678926107781868876 Sorry about the photo, but my school is about to set the alarm, and I need to get out of here. Hopefully it is visible.

Thanks,
Chris

kq6up said:

Homework Statement



Find the limit of: ##\frac { \Gamma (n+\frac { 3 }{ 2 } ) }{ \sqrt { n } \Gamma (n+1) } ## as ##n\rightarrow \infty ##.

Homework Equations



##\Gamma (p+1)=p^{ p }e^{ -p }\sqrt { 2\pi p } ##

The Attempt at a Solution



Mathematica and wolfram Alpha gave the limit as 1. My solution was ##\frac{1}{\sqrt{e}}##. My work is here https://plus.google.com/u/0/1096789...6080238892798007794&oid=109678926107781868876 Sorry about the photo, but my school is about to set the alarm, and I need to get out of here. Hopefully it is visible.

Thanks,
Chris

You wrote
[tex] \Gamma (p+1)=p^{ p }e^{ -p }\sqrt { 2\pi p }\; \Longleftarrow \; \text{FALSE!} [/tex]
Perhaps you mean ##\Gamma(p+1) \sim p^p e^{-p} \sqrt{2 \pi p}, ##, where ##\sim## means "is asymptotic to". That is a very different type of statement.

Anyway, I (or, rather, Maple) get a different final answer (=1), using the asymptotic result above. Somewhere you must have made an algebraic error.
 
Last edited:
  • #3
Sorry, yes I meant asymptotic to. Were you able to see my solution via the google plus link? Mary Boas' manual also gives a limit of one. Our solutions start out the same, but I make some approximations that seem not to be justified in her steps. However, I am unable to follow her steps. I can post that too if you are interested in taking a look at her solution.

Thanks,
Chris
 
  • #4
kq6up said:
Sorry, yes I meant asymptotic to. Were you able to see my solution via the google plus link? Mary Boas' manual also gives a limit of one. Our solutions start out the same, but I make some approximations that seem not to be justified in her steps. However, I am unable to follow her steps. I can post that too if you are interested in taking a look at her solution.

Thanks,
Chris

I don't have her book, and cannot really follow your screenshot. Anyway, if ##r(n)## is your ratio, and using ##\Gamma(p+1) \sim c p^{p+1/2} e^{-p}##, the numerator ##N(n)## has asymptotic form
[tex] N(n) = \Gamma(n+3/2) \sim c (n+1/2)^{n+1/2 + 1/2} e^{-(n+1/2)} = c (n+1/2)^{n+1} e^{-n} e^{-1/2}[/tex]
The denominator ##D(n)## has the asymptotic form
[tex] D(n) = \sqrt{n} \: \Gamma(n+1) \sim n^{1/2} c n^{n+1/2} e^{-n} = c n^{n+1} e^{-n} [/tex]
Therefore,
[tex] r(n) \sim \frac{(n+1/2)^{n+1} e^{-n} e^{-1/2}}{n^{n+1} e^{-n}} = \left( 1 + \frac{1}{2} \frac{1}{n} \right)^{n+1} e^{-1/2}[/tex]
Since ##\lim \,(1 + a/n)^{n+1} = \lim \,(1+a/n)^n = e^a##, we are done: ## \lim r(n) = 1##.
 
  • Like
Likes kq6up
  • #5
That is pretty much her solution. However, I can actually follow yours better. I made an approximation that was not justified.

Thanks,
Chris
 

Related to What is the limit of Gamma function as n approaches infinity?

1. What is Sterling's Approximation?

Sterling's Approximation, also known as Sterling's formula or Sterling's approximation, is a mathematical formula used to approximate factorials of large numbers.

2. How is Sterling's Approximation derived?

Sterling's Approximation is derived from the Stirling's series, a mathematical series that approximates the gamma function. The gamma function is a generalization of the factorial function.

3. What is the purpose of Sterling's Approximation?

The purpose of Sterling's Approximation is to provide a quick and easy way to approximate factorials of large numbers, which can be difficult and time-consuming to calculate directly.

4. What is the formula for Sterling's Approximation?

The formula for Sterling's Approximation is: n! ≈ √(2πn) * (n/e)^n. This formula can also be written as: n! ≈ (√(2πn) / e)^n.

5. What are the limitations of Sterling's Approximation?

Sterling's Approximation is not exact and can only provide an approximation of factorials. It is most accurate for large values of n, but becomes less accurate for smaller values. It also does not work well with fractions or negative numbers.

Similar threads

  • Calculus and Beyond Homework Help
Replies
24
Views
1K
  • Calculus and Beyond Homework Help
Replies
14
Views
1K
  • Calculus and Beyond Homework Help
Replies
8
Views
951
  • Calculus and Beyond Homework Help
Replies
3
Views
2K
Replies
13
Views
1K
  • Calculus and Beyond Homework Help
Replies
12
Views
6K
  • Calculus and Beyond Homework Help
Replies
8
Views
11K
  • Calculus and Beyond Homework Help
Replies
5
Views
2K
  • Calculus and Beyond Homework Help
Replies
5
Views
2K
  • Calculus and Beyond Homework Help
Replies
12
Views
937
Back
Top